www.vorhilfe.de
- Förderverein -
Der Förderverein.

Gemeinnütziger Verein zur Finanzierung des Projekts Vorhilfe.de.
Hallo Gast!einloggen | registrieren ]
Startseite · Mitglieder · Impressum
Forenbaum
^ Forenbaum
Status VH e.V.
  Status Vereinsforum

Gezeigt werden alle Foren bis zur Tiefe 2

Navigation
 Startseite...
 Suchen
 Impressum
Das Projekt
Server und Internetanbindung werden durch Spenden finanziert.
Organisiert wird das Projekt von unserem Koordinatorenteam.
Hunderte Mitglieder helfen ehrenamtlich in unseren moderierten Foren.
Anbieter der Seite ist der gemeinnützige Verein "Vorhilfe.de e.V.".
Partnerseiten
Weitere Fächer:

Open Source FunktionenplotterFunkyPlot: Kostenloser und quelloffener Funktionenplotter für Linux und andere Betriebssysteme
Forum "Schul-Analysis" - richtig Bewiesen?
richtig Bewiesen? < Analysis < Oberstufe < Schule < Mathe < Vorhilfe
Ansicht: [ geschachtelt ] | ^ Forum "Schul-Analysis"  | ^^ Alle Foren  | ^ Forenbaum  | Materialien

richtig Bewiesen?: Frage (beantwortet)
Status: (Frage) beantwortet Status 
Datum: 14:03 Fr 18.06.2004
Autor: AndreHarrweg

Irgendwie habe ich das gefühl als ob ich bei dem Beweis eines spziellen grenzwertsatzes etwas falsch gemacht habe. Kann mal jemand nachsehen der Ahnung hat ?

Bewiesen werden soll
[mm] \limes_{n \to \infty}(k+a_n)=k+\limes_{n \to \infty}a_n [/mm]

Da k eine Kostante Folge ist gilt:
[mm] \limes_{n \to \infty}(b_n+a_n)=\limes_{n \to \infty}(k)+\limes_{n \to \infty}(a_n)=k+ \limes_{n \to \infty}a_n [/mm]

Beweis:
Voraussetzung:
[mm] \limes_{n \to \infty}k=k, [/mm] k ist konstante Folge
[mm] \limes_{n \to \infty}a_n=\alpha [/mm]

dann gilt auch : [mm] |a_n-\alpha|<\epsilon n>N_1_(_\epsilon_) [/mm]

Es folgt daraus:
[mm] |(k+a_n)-\alpha|=|k+(a_n-\alpha)|\le|a_n-\alpha|<\epsilon [/mm]

d.h. es gilt
[mm] \left\{ k+a_n|n>N _( _\epsilon_) \right\} \subset\ U_\epsilon (\alpha) [/mm]


        
Bezug
richtig Bewiesen?: Antwort
Status: (Antwort) fertig Status 
Datum: 14:56 Fr 18.06.2004
Autor: Marcel

Hallo!

> Irgendwie habe ich das gefühl als ob ich bei dem Beweis
> eines spziellen grenzwertsatzes etwas falsch gemacht habe.
> Kann mal jemand nachsehen der Ahnung hat ?

Dann hoffen wir mal, dass ich Ahnung habe! ;-)
  

> Bewiesen werden soll
>  [mm]\limes_{n \to \infty}(k+a_n)=k+\limes_{n \to \infty}a_n [/mm]

>

Genau! Für eine Konstante $k$ und eine konvergente Folge [m](a_n)_{n \in \IN}[/m].

>
> Da k eine Kostante Folge ist gilt:
>  [mm]\limes_{n \to \infty}(b_n+a_n)=\limes_{n \to \infty}(k)+\limes_{n \to \infty}(a_n)=k+ \limes_{n \to \infty}a_n [/mm]

Wenn du es hier doch schon so schreibst, warum schreibst du den Beweis nicht einfach so:
Es sei [m](a_n)_{n \in \IN}[/m] eine konvergente Folge mit
1.) [m]a:=\limes_{n \to \infty}{a_n}[/m]
und es sei [m](b_n)_{n \in \IN}[/m] definiert durch [m]b_n:=k[/m]  [m]\forall n \in \IN[/m], wobei $k$ eine Konstante sei.
Dann ist  [m](b_n)_{n \in \IN}[/m] konvergent mit
2.) [m]\limes_{n \to \infty}{b_n}=k[/m]
(Beweis dazu kann ich dir auch nachliefern, wenn du einen brauchst) und es gilt:
[m]\limes_{n \to \infty}{(k+a_n)}=\limes_{n \to \infty}{(b_n+a_n)}=(\limes_{n \to \infty}{b_n})+(\limes_{n \to \infty}{a_n})=...[/m] (dies gilt vermutlich nach einem Satz aus der Vorlesung, ich ergänze ihn gleich noch; siehe (*)) und mit 1.) und 2.) folgt:
[m]...=k+\limes_{n \to \infty}{a_n}[/m].

(*) Vorausgesetzt, ihr wißt:
Sind [m](c_n)_{n \in \IN}[/m], [m](d_n)_{n \in \IN}[/m] konvergente Folgen, dann gilt:
[m]\limes_{n \to \infty}{(c_n+d_n)}=(\limes_{n \to \infty}{c_n})+(\limes_{n \to \infty}{d_n})[/m].

Na gut, aber egal. :-)

>
> Beweis:
>  Voraussetzung:
> [mm]\limes_{n \to \infty}k=k,[/mm] k ist konstante Folge
>  [mm]\limes_{n \to \infty}a_n=\alpha [/mm]

Hier würdest du besser schreiben:
Es gilt: [mm]\limes_{n \to \infty}k=k,[/mm] (konstante Folge) und es sei [mm]\alpha:=\limes_{n \to \infty}a_n[/mm]

>  
> dann gilt auch : [mm]|a_n-\alpha|<\epsilon n>N_1_(_\epsilon_) [/mm]

Mich stört etwas die Schreibweise. Normalerweise schreibt man es eher so:
Sei [mm] $\epsilon [/mm] > 0$ gegeben. Da [mm]\alpha=\limes_{n \to \infty}a_n[/mm] gilt, existiert ein [mm] $N_1_(_\epsilon_)$, [/mm] so dass für alle [m]n>N_1_(_\epsilon_)[/m]:
[mm] $|a_n-\alpha|<\epsilon$. [/mm]
Aber nun gut. Wenn du weißt, wie das zu lesen ist, ist deins in Ordnung! :-)
(Ergänze aber unbedingt, dass [mm] $\epsilon [/mm] > 0$ ist!)
  

>
> Es folgt daraus:
>  
> [mm]|(k+a_n)-\alpha|=|k+(a_n-\alpha)|\le|a_n-\alpha|<\epsilon [/mm]

Das stimmt so nicht! Der Grenzwert von [mm] $k+a_n$ [/mm] soll doch [mm] $k+\alpha$ [/mm] sein und nicht [mm] $\alpha$. [/mm] Das heißt, du mußt das ändern:
[mm]|(k+a_n)-(k+\alpha)|=|k+a_n-k-\alpha|=|a_n-\alpha| < \epsilon [/mm], und dann stimmt die Abschätzung auch (für alle [m]n>N_1_(_\epsilon_)[/m]). :-)

Vor allem diese Stelle bei dir:

> [mm] |k+(a_n-\alpha)|\le|a_n-\alpha|<\epsilon [/mm]

macht ja keinen Sinn. Wie kommst du darauf?
  

> d.h. es gilt
>  [mm]\left\{ k+a_n|n>N _( _\epsilon_) \right\} \subset\ U_\epsilon (\alpha) [/mm]
>  

Das musst du dann auch ändern:
[mm]\left\{ k+a_n|n>N _( _\epsilon_) \right\} \subset\ U_\epsilon (k+\alpha)[/mm]

Ich hoffe, dass dir das alles klar ist, denn ich muss gleich leider weg:
[mussweg]

[winken]

Viele Grüße
Marcel

Bezug
                
Bezug
richtig Bewiesen?: Ergänzung!
Status: (Mitteilung) Reaktion unnötig Status 
Datum: 15:16 Fr 18.06.2004
Autor: Marcel

Hallo Andre,
ich habe nach 2.) noch etwas hinzugefügt. Vorher stand dort:
[m]\limes_{n \to \infty}{(b_n+a_n)}=...[/m], jetzt steht dort:
[m]\limes_{n \to \infty}{(k+a_n)}=\limes_{n \to \infty}{(b_n+a_n)}=...[/m]

Ist irgendwie besser, wenn man das so schreibt, dann ist auch klar, dass man zuerst nur ausnutzt, dass [mm] $b_n=k$ [/mm] für alle $n [mm] \in \IN$ [/mm] gilt (nach Definition von [m](b_n)_{n \in \IN}[/m]).

Viele Grüße
Marcel

Bezug
                        
Bezug
richtig Bewiesen?: Ergänzung!
Status: (Frage) beantwortet Status 
Datum: 15:54 Fr 18.06.2004
Autor: AndreHarrweg

OK! also irgendwie fehlt mir im moment da noch ein wenig das verständnis des ganzen.

Ist folgendes also als Beweis für

[mm] \limes_{n \to \infty}(k+a_n)=k+\limes_{n \to \infty}a_n [/mm]

vollkommen ausreichend ?

Es sei:
[mm] (a_n) [/mm] n [mm] \in\IN\sub [/mm]
k eine Konstante
und es gilt:
[mm] \limes_{n \to \infty}(a_n+b_n)=\limes_{n \to \infty}a_n+\limes_{n \to \infty}b_n [/mm]
dann folgt:
[mm] \limes_{n \to \infty}(a_n+b_n)=\limes_{n \to \infty}a_n+\limes_{n \to \infty}b_n=\limes_{n \to \infty}k+\limes_{n \to \infty}(a_n)=k+\limes_{n \to \infty}(a_n) [/mm]

Müsste man die Abschätzung mit einbeziehen ?

Bezug
                                
Bezug
richtig Bewiesen?: Ergänzung!
Status: (Antwort) fertig Status 
Datum: 19:04 Fr 18.06.2004
Autor: Marcel

Hallo Andre,
erstmal entschuldige, dass ich von einer Vorlesung gesprochen hatte, wir sind ja noch im Schülerforum. ;-)

Nun zu dem Beweis:
Wenn du den Beweis mit der Abschätzung (also Definition von Konvergenz) machen willst, dann nimmst du deinen Beweis, den ich ja nun korrigiert habe. Ich habe das ganze nur etwas (meiner Ansicht nach) verständlicher formuliert und auch noch korrigiert, also:

Behauptung:
Ist [mm] $(a_n)_{n \in \IN}$ [/mm] eine konvergente Folge und ist $k [mm] \in \IR$ [/mm] eine feste Zahl, dann gilt für die Folge [mm] $(c_n)_{n \in \IN}$ [/mm] definiert durch [m]c_n:=k+a_n[/m]:
[mm] $(c_n)_{n \in \IN}$ [/mm] ist konvergent und es gilt:
[m]\limes_{n \to \infty}c_n=k+\limes_{n \to \infty}a_n[/m]

Das wollen wir beweisen. Wir führen zwei Beweise durch (einer davon würde aber vollkommen genügen!) Also:

Beweis mit [mm] $\epsilon-$Kriterium: [/mm]
Sei [mm] $\epsilon [/mm] > 0$ gegeben und sei [mm]\alpha:=\limes_{n \to \infty}a_n[/mm] [mm] ($(a_n)_{n \in \IN}$ [/mm] ist ja konvergent, beachte dies!).
Da [mm] $(a_n)_{n \in \IN}$ [/mm] gegen $a$ konvergiert, existiert ein [m]N_\epsilon[/m], so dass gilt:
(*) [mm]|a_n-\alpha|<\epsilon[/mm]   für alle [mm] $n>N_\epsilon$ [/mm]

Weiter gilt dann auch:
[m]|c_n-(k+\alpha)|=|k+a_n-(k+\alpha)|=|a_n-\alpha|[/m], und wenn wir (*) beachten, dann gilt also:
[m]|c_n-(k+\alpha)|=|k+a_n-(k+\alpha)|=|a_n-\alpha| < \epsilon[/m], und zwar für alle [mm] $n>N_\epsilon$ [/mm] (das steht ja auch in (*)!).

Also gilt:
[mm]\left\{ c_n | n>N _\epsilon \right\}=\left\{ k+a_n | n > N _\epsilon \right\} \subset\ U_\epsilon (k+\alpha) [/mm], also die Behauptung!

So würde ich den Beweis über das [mm] $\epsilon-$Kriterium [/mm] hinschreiben. Beachte, dass dieses [mm] $k+a_n$ [/mm] ja eigentlich eine Folge ist, deshalb habe ich die Folge [mm] $(c_n)_{n \in \IN}$ [/mm] ja so definiert! :-)

Nun kann man, wenn man gewisse Aussagen über Grenzwerte von Folgen schon einmal bewiesen hat, diese ausnutzen.
Ich gehe mal davon aus, dass ihr folgendes bewiesen habt:
1.) Sind [mm] $(r_n)_{n \in \IN}$, $(s_n)_{n \in \IN}$ [/mm] konvergente Folgen, dann gilt:
[mm]\limes_{n \to \infty}(r_n+s_n)=(\limes_{n \to \infty}r_n)+(\limes_{n \to \infty}s_n)[/mm]
2.) Ist [mm] $(t_n)_{n \in \IN}$ [/mm] eine konstante Folge, d.h. [mm] $t_n=c$ [/mm] für ein [m]c \in \IR[/m] und für alle $n [mm] \in \IN$, [/mm] dann gilt:
[mm] $\limes_{n \to \infty}t_n=c$. [/mm]

Falls nicht, dann nimm den Beweis über das [mm] $\epsilon-$Kriterium [/mm] (oder beweise zunächst mit dem [m]\epsilon-[/m]Kriterium 1.) und 2.) ;-)).
Falls aber doch, so kannst du hiermit natürlich auch den Beweis führen:
Zunächst zur Erinnerung nochmal die Behauptung:
Behauptung:
Ist [mm] $(a_n)_{n \in \IN}$ [/mm] eine konvergente Folge und ist $k [mm] \in \IR$ [/mm] eine feste Zahl, dann gilt für die Folge [mm] $(c_n)_{n \in \IN}$ [/mm] definiert durch [m]c_n:=k+a_n[/m]:
[mm] $(c_n)_{n \in \IN}$ [/mm] ist konvergent und es gilt:
[m]\limes_{n \to \infty}c_n=k+\limes_{n \to \infty}a_n[/m]

Und nun gehts los:
2. Beweis:
Es sei [mm] $(b_n)_{n \in \IN}$ [/mm] definiert durch [mm] $b_n:=k$, [/mm] wobei $k [mm] \in \IR$ [/mm] fest ist. Wegen 2.) ist [mm] $(b_n)_{n \in \IN}$ [/mm] konvergent mit
3.) [m]\limes_{n \to \infty}b_n=k[/m].
Nach Voraussetzung ist auch [mm] $(a_n)_{n \in \IN}$ [/mm] konvergent. Offenbar gilt für alle $n [mm] \in \IN$: [/mm]
(I) [mm] $c_n=b_n+a_n$. [/mm]
Weil nun [mm] $(a_n)_{n \in \IN}$ [/mm] und [mm] $(b_n)_{n \in \IN}$ [/mm] konvergent sind, können wir bei (I) 1.) anwenden und erhalten:
[mm]\limes_{n \to \infty}c_n=\limes_{n \to \infty}(b_n+a_n)=(\limes_{n \to \infty}b_n)+(\limes_{n \to \infty}a_n)[/mm]

Mit 3.) und weil [mm] $b_n=k$ [/mm] für alle $n [mm] \in \IN$ [/mm] gilt, folgt daraus nun:
[mm]\limes_{n \to \infty}c_n=\limes_{n \to \infty}(k+a_n) =\limes_{n \to \infty}(b_n+a_n) =(\limes_{n \to \infty}b_n)+(\limes_{n \to \infty}a_n) =k+\limes_{n \to \infty}a_n[/mm]

Ende des zweiten Beweises!

> OK! also irgendwie fehlt mir im moment da noch ein wenig
> das verständnis des ganzen.
>  
> Ist folgendes also als Beweis für
>  
> [mm]\limes_{n \to \infty}(k+a_n)=k+\limes_{n \to \infty}a_n [/mm]
>  
>
> vollkommen ausreichend ?
>  
> Es sei:
>   [mm](a_n)[/mm] n [mm]\in\IN\sub[/mm]
> k eine Konstante
>  und es gilt:
>  [mm]\limes_{n \to \infty}(a_n+b_n)=\limes_{n \to \infty}a_n+\limes_{n \to \infty}b_n [/mm]
>  
> dann folgt:
>  [mm]\limes_{n \to \infty}(a_n+b_n)=\limes_{n \to \infty}a_n+\limes_{n \to \infty}b_n=\limes_{n \to \infty}k+\limes_{n \to \infty}(a_n)=k+\limes_{n \to \infty}(a_n) [/mm]

Ja, fast. Du mußt halt erwähnen, dass [mm] $(b_n)_{n \in \IN}$ [/mm] eine konstante Folge ist, die nur den Wert $k$ annimmt, und somit konvergiert. Weil dann nämlich [mm] $(a_n)_{n \in \IN}$ [/mm] und [mm] $(b_n)_{n \in \IN}$ [/mm] konvergieren ([m](a_n)_{n \in \IN}[/m] konvergiert ja nach Voraussetzung), darfst du die Regel (welche ich hier mit 1.) bezeichnet habe) anwenden. Das ist wesentlich, wenn man den Beweis über die Regel 1.) führen will. Wir brauchen dort ja die Voraussetzung der Konvergenz der beiden Folgen.

(Wenn man unterstellt, dass dir bekannt ist, dass
[mm] $\limes_{n \to \infty}k=k$ [/mm] nichts anderes bedeutet, als das eine konstante Folge, die nur den Wert $k$ annimmt, auch den Grenzwert $k$ hat und somit konvergiert, kannst du das auch so stehen lassen! Ist immer etwas schwierig für mich, zu beurteilen, ob dir das klar ist...)


> Müsste man die Abschätzung mit einbeziehen ?

  
Ich wüßte nicht, warum. Die Abschätzung (ich denke, du meinst die mit dem [mm] $\epsilon$?) [/mm] macht man ja bei dem [m]\epsilon-[/m]Kriterium (also bei dem ersten Beweis hier!).
Bei der Regel 1.) brauchen wir das gar nicht, sondern, wie gesagt, nur die Voraussetzung der Konvergenz der beiden Folgen!

Viele Grüße
Marcel

Bezug
                                        
Bezug
richtig Bewiesen?: Ergänzung 2
Status: (Mitteilung) Reaktion unnötig Status 
Datum: 19:07 Fr 18.06.2004
Autor: Marcel

Achja, unsere Folgen hier sind immer reellwertige Folgen (komplexwertige Folgen etc. betrachtet ihr ja vermutlich nicht!). Ich habe das nicht extra dazugeschrieben, will es aber an dieser Stelle noch ergänzen!

Viele Grüße
Marcel

Bezug
Ansicht: [ geschachtelt ] | ^ Forum "Schul-Analysis"  | ^^ Alle Foren  | ^ Forenbaum  | Materialien


^ Seitenanfang ^
ev.vorhilfe.de
[ Startseite | Mitglieder | Impressum ]